LSAT and Law School Admissions Forum

Get expert LSAT preparation and law school admissions advice from PowerScore Test Preparation.

User avatar
 Dave Killoran
PowerScore Staff
  • PowerScore Staff
  • Posts: 5853
  • Joined: Mar 25, 2011
|
#85194
Complete Question Explanation
(The complete setup for this game can be found here: lsat/viewtopic.php?f=332&t=3599)

The correct answer choice is (B)

Answer choice (A) is incorrect because according to the fourth rule, J must evaluate X during week 4.

Answer choice (B) is the correct answer.

Answer choice (C) is incorrect because from the third rule R cannot evaluate two introductory books consecutively, and in this answer R evaluates G and H consecutively.

Answer choice (D) is incorrect because from the first rule R cannot evaluate an introductory textbook until J has evaluated it, and in this answer R evaluates F before J evaluates F.

Answer choice (E) is incorrect because from the second rule J cannot evaluate an advanced textbook until R has evaluated it, and in this answer J evaluates Y before R evaluates Y.

Get the most out of your LSAT Prep Plus subscription.

Analyze and track your performance with our Testing and Analytics Package.